Diễn Đàn MathScopeDiễn Đàn MathScope
  Diễn Đàn MathScope
Ghi Danh Hỏi/Ðáp Thành Viên Social Groups Lịch Ðánh Dấu Ðã Ðọc

Go Back   Diễn Đàn MathScope > Sơ Cấp > Tài Liệu > Đề Thi > Đề Thi HSG Cấp Tỉnh ở Việt Nam

News & Announcements

Ngoài một số quy định đã được nêu trong phần Quy định của Ghi Danh , mọi người tranh thủ bỏ ra 5 phút để đọc thêm một số Quy định sau để khỏi bị treo nick ở MathScope nhé !

* Nội quy MathScope.Org

* Một số quy định chung !

* Quy định về việc viết bài trong diễn đàn MathScope

* Nếu bạn muốn gia nhập đội ngũ BQT thì vui lòng tham gia tại đây

* Những câu hỏi thường gặp

* Về việc viết bài trong Box Đại học và Sau đại học


Trả lời Gởi Ðề Tài Mới
 
Ðiều Chỉnh Xếp Bài
Old 18-09-2019, 11:00 AM   #16
Le khanhsy
Super Moderator
 
Tham gia ngày: Oct 2017
Bài gởi: 48
Thanks: 52
Thanked 57 Times in 30 Posts
Trích:
Nguyên văn bởi Le khanhsy View Post
BẤT ĐẲNG THỨC- HỆ PHƯƠNG TRÌNH
[*][Bình Dương] Cho $x,\,y,\,z$ thỏa mãn $xy+yz+zx=1$, tìm giá trị nhỏ nhất của$$ T=\frac{1}{x^{2}+1}+\frac{1}{y^{2}+1}+\frac{1}{z^{ 2}+1}. $$
Còn lại 1 bài này duy nhất nhưng theo em nghĩ thì có max không phải min.
[RIGHT][I][B]Nguồn: MathScope.ORG[/B][/I][/RIGHT]
 
Le khanhsy is offline   Trả Lời Với Trích Dẫn
The Following User Says Thank You to Le khanhsy For This Useful Post:
MATHSCOPE (20-09-2019)
Old 18-09-2019, 11:53 AM   #17
Le khanhsy
Super Moderator
 
Tham gia ngày: Oct 2017
Bài gởi: 48
Thanks: 52
Thanked 57 Times in 30 Posts
Trích:
Nguyên văn bởi Le khanhsy View Post
Viết lại phương trình như sau $$ x^{3}+x^{2}-3 x-2- (x+2)=2\sqrt{x+2}-(x+2),$$ hay là $$(x^2-4)(x+1)=\dfrac{4-x^2}{2\sqrt{x+2}+(x+2)},$$ $$(x^2-4)\left[x+1+\dfrac{1}{2\sqrt{x+2}+(x+2)} \right]=0.$$ Bỏ qua nghiệm tầm thường $x=\pm2$. Xét phương trình $$t^2-1+\dfrac{1}{2t+t^2}=0,\,\ 0\le t\le 2,$$ hay là $$(t^2+t-1)^2=0.$$ Giải phương trình trinh thì thu được $x=-\dfrac{1+\sqrt{5}}{2}.$ Vậy tập nghiệm phương trình là $S=\left\{-2,-\dfrac{1+\sqrt{5}}{2},2\right\}$.
Cách 2
Đặt $x=2 cost$ với $t\in \left[0,\pi\right]$. Phương trình viết lại như sau
$$cos(3t)+cos(2t)=2\left|cos\dfrac{t}{2}\right|,$$
hay là
$$cos\left(\dfrac{5t}{2}\right).cos\left(\dfrac{t} {2}\right)=cos\dfrac{t}{2},$$
Trường hợp $cos\left(\dfrac{t}{2}\right)=0$ ta được $cost=-1$ hay $x=-2$.
Trường hợp $cos\left(\dfrac{5t}{2}\right)=1$ ta được $t=0;\dfrac{4\pi}{5}$, hay $x=\left\{2;2cos\left(\dfrac{4\pi}{5}\right)\right \}$.
[RIGHT][I][B]Nguồn: MathScope.ORG[/B][/I][/RIGHT]
 
Le khanhsy is offline   Trả Lời Với Trích Dẫn
The Following User Says Thank You to Le khanhsy For This Useful Post:
MATHSCOPE (20-09-2019)
Old 18-09-2019, 03:07 PM   #18
MathForLife
+Thành Viên+
 
Tham gia ngày: Sep 2010
Đến từ: CT force
Bài gởi: 731
Thanks: 603
Thanked 425 Times in 212 Posts
[*][Quảng Bình] Cho $p$ là số nguyên tố lớn hơn $3$ và $n=2^{2p}-1$. Chứng minh rằng, $n$ có ít nhất ba ước nguyên tố phân biệt và $$n\mid\left(2^n-8\right).$$


$n=2^{2p}-1=(2^{p-1}+2^{p-2}+...+2+1).3.(2^{p-1}-2^{p-2}+...-2+1)=A.3.B$. Ta có $$A\equiv 1, B\equiv 1, -1 \text{(mod 3)}$$ và $$A\neq B$$ nên $A$ và $B$ sẽ có các ước nguyên tố khác nhau và khác $3$. Vậy $n$ có ít nhất $3$ ước nguyên tố.

$n=2^{2p}-1 \equiv 3 \text{(mod p)}\Rightarrow tp+3=2^{2p}-1=n$ với $t\equiv 0 \text{(mod 2)}$

Suy ra $2^{n}-8=2^{tp+3}-8=8(2^{tp}-1)$. Mà $(2^{tp}-1,2^{2p}-1)=2^{(tp,2p)-1}=2^{2p}-1$ nên ta có đpcm.
[RIGHT][I][B]Nguồn: MathScope.ORG[/B][/I][/RIGHT]
 
__________________

thay đổi nội dung bởi: MathForLife, 18-09-2019 lúc 03:20 PM
MathForLife is offline   Trả Lời Với Trích Dẫn
The Following User Says Thank You to MathForLife For This Useful Post:
MATHSCOPE (20-09-2019)
Old 18-09-2019, 04:24 PM   #19
MathForLife
+Thành Viên+
 
Tham gia ngày: Sep 2010
Đến từ: CT force
Bài gởi: 731
Thanks: 603
Thanked 425 Times in 212 Posts
[*][Bắc Giang] Tìm tất cả các số nguyên dương $m$ có tính chất: nếu $a$ và $b$ là các ước số nguyên dương của $m$ và $\gcd\left(a,\,b\right)=1$ thì $a+b-1$ cũng chia hết $m$.


Đặt $n=p_{1}^{\alpha_{1}}...p_{k}^{\alpha_{k}}$. Chọn $a=p_{1}$, $b=p_{2}^{\alpha_{2}}...p_{k}^{\alpha_{k}}$. Khi đó $(a,b)=1$, $a|n$, $b|n$.

$a+b-1>b$ cũng là ước của $m$ nên $a+b-1=a^{k}b$. Mà $a^{k}b>ab>a+b-1$ với mọi $a,b>1$ nên ta có $a$ hoặc $b$ bằng 1. Suy ra $m$ có dạng $p^{k}$ với $p$ nguyên tố.
[RIGHT][I][B]Nguồn: MathScope.ORG[/B][/I][/RIGHT]
 
__________________
MathForLife is offline   Trả Lời Với Trích Dẫn
The Following User Says Thank You to MathForLife For This Useful Post:
MATHSCOPE (20-09-2019)
Old 18-09-2019, 05:09 PM   #20
Thụy An
+Thành Viên+

 
Tham gia ngày: Oct 2017
Bài gởi: 93
Thanks: 1
Thanked 68 Times in 45 Posts
Trích:
Nguyên văn bởi MATHSCOPE View Post
[*][Bắc Giang] Tìm tất cả các số nguyên dương $m$ có tính chất: nếu $a$ và $b$ là các ước số nguyên dương của $m$ và $\gcd\left(a,\,b\right)=1$ thì $a+b-1$ cũng chia hết $m$.
Ta viết $n=2^kN$ với $N$ là số lẻ, để ý rằng nếu $a,\,b$ đều là ước của $N$ và $\gcd (a,\,b)=1$ thì $a\,b$ cũng là ước của $n$ vì thế $a+b-1$ cũng là ước lẻ của $n$ và do đó $a+b-1$ là ước của $N$. Tức là $N$ cũng có tính chất như $n$.

Giờ ta giả sử $p$ là ước nguyên tố bé nhất của $N$ và $v_p(N)=l$, khi đó đặt $\frac{N}{p^l}=N'$ thì $N'\mid N,\,p\mid N$ và $\gcd(p,\,N')=1$ nên\[(p+N'-1)\mid N.\]Nếu $N'$ có một ước nguyên tố là $q$ nào đó, khi đó do vai trò của $p$ nên $q\nmid (p-1)$ vậy nên $v_q(p-1)=0<v_q(N')$ theo tính chất phi Archimedean có\[{v_q}\left( {p + N' - 1} \right) = {v_q}\left( {p - 1} \right) = 0.\]Điều này cho thấy là tồn tại $t\in\mathbb N^*,\,t\le l$ sao cho\[p + N' - 1 = {p^t},\;(1).\]Vì đang xét $N'>1$ nên $l\ge t> 1$, lại để ý là $2\le\frac{p+1}{2}\le p-1<q$ nên\[{v_q}\left( {{p^2} - 1} \right) = {v_q}\left( {p - 1} \right) + {v_q}\left( 2 \right) + {v_q}\left( {\frac{{p + 1}}{2}} \right) = 0.\]Do đó theo tính chất phi Archimedean lại có\[{v_q}\left( {{p^2} + N' - 1} \right) = {v_q}\left( {{p^2} - 1} \right) = 0.\]Tức là sẽ tồn tại $T\in\mathbb N^*$ thỏa $t<T\le l$ để\[{p^2} + N' - 1 = {p^T},\;(2).\]Từ $(1)$ và $(2)$ sẽ có\[p\left( {p - 1} \right) = {p^2} - p = {p^t}\left( {{p^{T - t}} - 1} \right).\]Lấy $v_p$ hai vế, ta có $t=1,\,T=2$ dẫn đến mâu thuẫn trong trường hợp đang xét là $N'=1$.

Vậy, $N=p^l$ tức $n=2^kp^l$ với $k,\,l\in\mathbb N$ và $p$ là số nguyên tố lẻ.
  1. Nếu $k$ hoặc $l=0$, rõ ràng ta thấy thỏa mãn.

  2. Nếu $k,\,l\in\mathbb N^*$, ta có $p+1=2+p-1$ chia hết $n$ và $\gcd (p+1,\,p)=1$ cho nên $p+1=2^u$ với $u\in\mathbb N$ và do $p\ge 3$ nên ta có $2\le u\le k$ để lại có $2^2+p-1=p+3$ là ước của $n$. Nếu $p\nmid (p+3)$ thì sẽ phải có số tự nhiên $v$ thỏa $v>u$ để $p+3=2^v$ lúc này có mâu thuẫn là $2=(p+3)-(p+1)=2^v-2^u$ chia hết cho $4$. Vậy, $p\mid (p+3)$ mà $p\ge 3$ nên $p<p+3\le 2p$ cho nên $p+3=2p$ tức $p=3$. Như vậy, $n$ không có ước nguyên tố lớn hơn $5$, cho nên không thể xảy đến $8+3-1$ và $9+2-1$ là ước của $n$. Vì thế, $k=2$ và $l=1$, tức là $n=12$.
Tóm lại: $n=p^k$ với $k\in\mathbb N$, $p$ là số nguyên tố hoặc $n=12$.
[RIGHT][I][B]Nguồn: MathScope.ORG[/B][/I][/RIGHT]
 
Thụy An is offline   Trả Lời Với Trích Dẫn
Old 18-09-2019, 05:42 PM   #21
MATHSCOPE
Administrator

 
Tham gia ngày: Nov 2007
Bài gởi: 30
Thanks: 110
Thanked 183 Times in 68 Posts
Trích:
Nguyên văn bởi Le khanhsy View Post
Còn lại 1 bài này duy nhất nhưng theo em nghĩ thì có max không phải min.
Đúng đề là giá trị lớn nhất em à
[RIGHT][I][B]Nguồn: MathScope.ORG[/B][/I][/RIGHT]
 
MATHSCOPE is offline   Trả Lời Với Trích Dẫn
The Following User Says Thank You to MATHSCOPE For This Useful Post:
Le khanhsy (19-09-2019)
Old 18-09-2019, 05:44 PM   #22
MATHSCOPE
Administrator

 
Tham gia ngày: Nov 2007
Bài gởi: 30
Thanks: 110
Thanked 183 Times in 68 Posts
Trích:
Nguyên văn bởi MathForLife View Post
Suy ra $m$ có dạng $p^{k}$ với $p$ nguyên tố.
Số 12 thỏa mãn.
[RIGHT][I][B]Nguồn: MathScope.ORG[/B][/I][/RIGHT]
 
MATHSCOPE is offline   Trả Lời Với Trích Dẫn
Old 18-09-2019, 07:49 PM   #23
Thụy An
+Thành Viên+

 
Tham gia ngày: Oct 2017
Bài gởi: 93
Thanks: 1
Thanked 68 Times in 45 Posts
Trích:
Nguyên văn bởi MATHSCOPE View Post
[Sài Gòn] Cho $P(x)$ là đa thức đơn khởi, hệ số thực có bậc là $2019$. Biết rằng $P(x)$ có $2019$ nghiệm thực không nguyên, đôi một phân biệt. Giả sử mỗi đa thức $P\left(2x^2-4x\right)$ và $P\left(4x-2x^2\right)$ đều có đúng $2692$ nghiệm thực phân biệt.
  1. Có bao nhiêu nghiệm của $P(x)$ trong $(-2;\,2)$?
  2. Chứng minh rằng tồn tại ba đa thức đồng bậc $A(x),\,B(x),\,C(x)$ sao cho $A(x)C(x)\ne B(x)$ với mọi $x\in (0;\,1)$ và \[P(x)=A(x)B(x)C(x).\]
Với mỗi tập con $S$ của $\mathbb R$ và đa thực $p$, ta ký hiệu $\cal N_p(S)$ là số nghiệm thực của $p(x)$ trên $S$.

Xét $f(x)=2x^2-4x$, ta có $f\left(\mathbb R\right)=\left[-2;\,+\infty\right)$ vì thế mỗi nghiệm $r$ của $P(f(x))$ sẽ ứng với duy nhất nghiệm là $f(r)\in \left(-2;\,+\infty\right)$ (do $f(r)\ne 0$). Đảo lại, mỗi nghiệm $x_r\in \left(-2;\,+\infty\right)$ của $P(x)$ sẽ ứng với đúng hai nghiệm của $P(f(x))$ là\[{r_1} = 1 + \sqrt {\frac{{2 + {x_r}}}{2}} ,\;\;\,{r_2} = 1 - \sqrt {\frac{{2 + {x_r}}}{2}} .\]Vậy là sẽ có đẳng thức\[2692 = {{\cal N}_{P\left( f \right)}}\left(\mathbb R \right) = 2{{\cal N}_P}\left( {\left( { - 2;{\mkern 1mu} + \infty } \right)} \right).\]Vậy là có được ${{\cal N}_P}\left( {\left( { - 2;{\mkern 1mu} + \infty } \right)} \right)=1346$, cũng tương tự để có ${{\cal N}_P}\left( {\left( { -\infty;\,2 } \right)} \right)=1346$, đến đây ta có được\[\begin{array}{l}
{{\cal N}_P}\left( {\left( { - \infty ;{\mkern 1mu} - 2} \right)} \right) = {{\cal N}_P}\left( \mathbb R \right) - {{\cal N}_P}\left( {\left( { - 2;{\mkern 1mu} + \infty } \right)} \right) = 673.\\
{{\cal N}_P}\left( {\left( {{\mkern 1mu} 2;{\mkern 1mu} + \infty } \right)} \right) = {{\cal N}_P}\left( \mathbb R \right) - {{\cal N}_P}\left( {\left( { - \infty ;{\mkern 1mu} \,2} \right)} \right) = 673.
\end{array}\]Cho nên số nghiệm trên $(-2;\,2)$ của $P(x)$ là\[{{\cal N}_P}\left( {\left( {{\mkern 1mu} - 2;\,2} \right)} \right) = {{\cal N}_P}\left( \mathbb R \right) - {{\cal N}_P}\left( {\left( { - \infty ;{\mkern 1mu} \, - 2} \right)} \right) - {{\cal N}_P}\left( {\left( {{\mkern 1mu} 2;{\mkern 1mu} + \infty } \right)} \right) = 673.\]
Bây giờ ta xét các đa thức\[A\left( x \right) = \prod\limits_{a \in {{\cal N}_P}\left( {\left( { - \infty ;{\mkern 1mu} \, - 2} \right)} \right)} {\left( {x - a} \right),\;\;\,} B\left( x \right) = \prod\limits_{b \in {{\cal N}_P}\left( {\left( { - 2;{\mkern 1mu} \,2} \right)} \right)} {\left( {x - b} \right),\;\;\,C\left( x \right) = \prod\limits_{c \in {{\cal N}_P}\left( {\left( {2;{\mkern 1mu} + \infty } \right)} \right)} {\left( {x - c} \right).} } \]Thế thì $\deg A=\deg B=\deg C=673$ và đồng thời có được\[P(x)=A(x)B(x)C(x).\]Thêm nữa với $a \in {{\cal N}_P}\left( {\left( { - \infty ;{\mkern 1mu} \, - 2} \right)} \right),\:b \in {{\cal N}_P}\left( {\left( { - 2;{\mkern 1mu} \,2} \right)} \right),\:c \in {{\cal N}_P}\left( {\left( {2;{\mkern 1mu} + \infty } \right)} \right)$ và $x\in (-1;\,1)$ thì\[\left| {x - b} \right| < \max \left\{ {\left| {x - a} \right|,\:\left| {x - c} \right|} \right\} < \left| {x - a} \right|\left| {x - c} \right|.\]
Ta hoàn chỉnh chứng minh.
[RIGHT][I][B]Nguồn: MathScope.ORG[/B][/I][/RIGHT]
 
Thụy An is offline   Trả Lời Với Trích Dẫn
The Following User Says Thank You to Thụy An For This Useful Post:
nmd2708 (23-09-2019)
Old 18-09-2019, 10:51 PM   #24
MathForLife
+Thành Viên+
 
Tham gia ngày: Sep 2010
Đến từ: CT force
Bài gởi: 731
Thanks: 603
Thanked 425 Times in 212 Posts
Trích:
Nguyên văn bởi MATHSCOPE View Post
Số 12 thỏa mãn.
Một chút sơ suất ạ. Xin chỉnh lại như sau.
[*][Bắc Giang] Tìm tất cả các số nguyên dương $m$ có tính chất: nếu $a$ và $b$ là các ước số nguyên dương của $m$ và $\gcd\left(a,\,b\right)=1$ thì $a+b-1$ cũng chia hết $m$.


Đặt $n=p_{1}^{\alpha_{1}}...p_{k}^{\alpha_{k}}$ trong đó $p_{i}<p_{j}$. Chọn $a=p_{1}$, $b=p_{2}^{\alpha_{2}}...p_{k}^{\alpha_{k}}$. Khi đó $(a,b)=1$, $a|n$, $b|n$.

$a+b-1>b$ cũng là ước của $m$ nên $a+b-1=a^{k}b$ hoặc $a+b-1=a^{k}t$ với $t$ là ước thực sự của $b$.

TH1: $a^{k}b>ab>a+b-1$ với mọi $a,b>1$ nên ta có $a$ hoặc $b$ bằng 1. Suy ra $m$ có dạng $p^{k}$ với $p$ nguyên tố.

TH2: Suy ra $a-1$ chia hết $t$ vậy $t=1$. Chọn $a=p_{1}^{\alpha_{1}-1}$. Khi đó $a+b-1=p_{1}^{\alpha_{1}-1}+p_{1}^{k}-p_{1}=p_{1}^{\alpha_{1}}$ hoặc bằng $p_{1}^{\alpha_{1}-1}$.Tất cả đều suy ra $b=1$ hoặc $\alpha_{1}=2$ và $p_{1}=2$ hay $n=12$.
[RIGHT][I][B]Nguồn: MathScope.ORG[/B][/I][/RIGHT]
 
__________________

thay đổi nội dung bởi: MathForLife, 19-09-2019 lúc 10:22 AM
MathForLife is offline   Trả Lời Với Trích Dẫn
The Following User Says Thank You to MathForLife For This Useful Post:
MATHSCOPE (20-09-2019)
Old 19-09-2019, 11:14 AM   #25
Ho Tung Quan
+Thành Viên+
 
Tham gia ngày: Jul 2016
Bài gởi: 15
Thanks: 1
Thanked 1 Time in 1 Post
Bình Dương - PTH

Tìm tất cả các hàm số $f:\mathbb{R}\rightarrow \mathbb{R}$ thỏa mãn: $f(x^2-y^2)=(x-y)[f(x)+f(y)], \forall x, y \in \mathbb{R}.$(1)
Lời giải
Thay $x=y$ vào (1), ta được: $f(0)=0.$
Thay $y=0$ vào (1), ta được: $f(x^2)=xf(x).$
Thay $x=0$ vào (1), ta được: $f(-y^2)=-yf(y).$
Từ đó suy ra $f$ là hàm số lẻ.
Đặt $a=x-y, b=x+y,$ từ (1) ta có: $f(ab)=a\left[f\left(\dfrac{a+b}{2}\right)+f\left(\dfrac{b-a}{2}\right)\right]$ $\rightarrow f\left(\dfrac{a+b}{2}\right)=\dfrac{f(ab)}{a}-f\left(\dfrac{b-a}{2}\right) (2)$
Do vế trái của (2) đối xứng nên: $\dfrac{f(ab)}{a}-f\left(\dfrac{b-a}{2}\right)=\dfrac{f(ab)}{b}-f\left(\dfrac{a-b}{2}\right).$
Từ đó: $f(ab)\left(\dfrac{1}{a}-\dfrac{1}{b}\right)=f\left(\dfrac{b-a}{2}\right)-f\left(\dfrac{a-b}{2}\right).$
Do đó: $\dfrac{f(ab)}{ab}=\dfrac{f(\frac{b-a}{2})}{\frac{b-a}{2}},$ (do $f$ lẻ).
Từ đó suy ra: $\dfrac{f(x)}{x}=\dfrac{f(y)}{y}=k, x\neq 0, y\neq 0.$
Do $f(0)=0$ nên $f(x)=kx, \forall x\in \mathbb{R}.$ Thử lại thấy thỏa.
[RIGHT][I][B]Nguồn: MathScope.ORG[/B][/I][/RIGHT]
 
Ho Tung Quan is offline   Trả Lời Với Trích Dẫn
The Following User Says Thank You to Ho Tung Quan For This Useful Post:
MATHSCOPE (20-09-2019)
Old 19-09-2019, 11:49 AM   #26
Le khanhsy
Super Moderator
 
Tham gia ngày: Oct 2017
Bài gởi: 48
Thanks: 52
Thanked 57 Times in 30 Posts
Chọn đội tuyển hồ chí minh 2019

Trích:
Cho các số thực $a,b,c,d$ thỏa mãn $a^2+b^2+c^2+d^2=1$. Chứng minh rằng
$$4(1-a)(1-b)\ge (c+d)^2$$
Áp dụng bất đẳng thức AM-GM cơ bản
$$2(c^2+d^2)\ge (c+d)^2.$$
Vì thế chúng ta cần chứng minh bất đẳng thức
$$4(1-a)(1-b)\ge 2(1-a^2-b^2),$$
hay
$$2(a+b-1)^2\ge 0. $$
Bất đẳng thức cuối cùng luôn đúng. Hoàn tất chứng minh
[RIGHT][I][B]Nguồn: MathScope.ORG[/B][/I][/RIGHT]
 
Le khanhsy is offline   Trả Lời Với Trích Dẫn
The Following User Says Thank You to Le khanhsy For This Useful Post:
MATHSCOPE (20-09-2019)
Old 19-09-2019, 12:58 PM   #27
Le khanhsy
Super Moderator
 
Tham gia ngày: Oct 2017
Bài gởi: 48
Thanks: 52
Thanked 57 Times in 30 Posts
Trích:
Nguyên văn bởi Le khanhsy View Post
BẤT ĐẲNG THỨC- HỆ PHƯƠNG TRÌNH
[*][Bình Dương] Cho $a,\,b,\,c$ thỏa mãn $ab+bc+ca=1$, tìm giá trị lớn nhất của$$ T=\frac{1}{a^{2}+1}+\frac{1}{b^{2}+1}+\frac{1}{c^{ 2}+1}. $$
Ta có theo gỉa thiết thì $(a+b)c=1-ab.$ Nếu 2 biến có tổng là 0 thì ta có ngay không tồng tại giả thiết. Vậy nên viết lại bất đẳng thức như sau $$T=\dfrac{1}{a^{2}+1}+\dfrac{1}{b^{2}+1}+\dfrac{1 }{ \left(\dfrac{1-ab}{a+b}\right)^2+1},$$ $$P=\dfrac{1}{a^{2}+1}+\dfrac{1}{b^{2}+1}+\dfrac{( a+b)^2}{(a^2+1)(b^2+1)}= \dfrac{2(a^2+b^2+ab+1)}{(a^2+1)(b^2+1)}=\dfrac{9}{ 4}- \dfrac{(3ab-1)^2+(a-b)^2}{2(a^2+1)(b^2+1)}.$$
Vậy nên $\max T=\dfrac{9}{4}.$ Hoàn tất bài toán
[RIGHT][I][B]Nguồn: MathScope.ORG[/B][/I][/RIGHT]
 
Le khanhsy is offline   Trả Lời Với Trích Dẫn
The Following User Says Thank You to Le khanhsy For This Useful Post:
MATHSCOPE (20-09-2019)
Old 19-09-2019, 06:49 PM   #28
Thụy An
+Thành Viên+

 
Tham gia ngày: Oct 2017
Bài gởi: 93
Thanks: 1
Thanked 68 Times in 45 Posts
Trích:
Nguyên văn bởi MATHSCOPE View Post
[Khánh Hòa] Chứng minh rằng với mỗi số nguyên dương $n$, đều tồn tại duy nhất một cặp số nguyên dương $(a,\,b)$ sao cho \[n = a + \frac{{\left( {a + b - 1} \right)\left( {a + b - 2} \right)}}{2}.\]
Ta sắp xếp các cặp số nguyên dương thành một dãy theo quy tắc: cặp $(1,\,1)$ đứng đầu dãy, cặp $\left(a,\,b\right)$ sẽ đứng trước cặp $\left(a',\,b'\right)$ nếu như $a+b>a'+b'$ hoặc là $a+b=a'+b'$ nhưng $a<a'$.

Việc sắp xếp thành một dãy bằng quy tắc đó, sẽ tạo nên một song ánh $f:\,\left(\mathbb N^*\right)^2\to\mathbb N^*$ cho tương ứng mỗi cặp số nguyên dương với thứ tự của nó trong dãy.

Giờ, ta đi tính $f\left((a,\,b)\right)$ với $(a,\,b)\ne (1,\,1)$ bằng cách đếm số các cặp đứng trước nó. Với một cặp $(u,\,v)$ đứng trước $(a,\,b)$, thì có hai khả năng
  1. Nếu $u+v<a+b$, đặt $a+b-1-u-v=w$ ta có $u+v+w=a+b-1$, ta quy về bài toán chia $a+b-1$ cái kẹo cho $3$ đứa trẻ sao cho mỗi đứa có ít nhất một cái. Và ta có kết quả là $\dbinom{a+b-1}{2}$.

  2. Nếu $u+v=a+b$, khi đó do $u\in\mathbb N^*$ và $u<a$ nên có đúng $a-1$ cặp $(u,\,v)$.
Như vậy, đứng trước cặp $(a,\,b)$ có đúng $ \dbinom{a+b-1}{2} +a-1$ cặp, cho nên\[f\left( {(a,{\mkern 1mu} b)} \right) =1+\dbinom{a+b-1}{2} +a-1= a + \frac{{\left( {a + b - 1} \right)\left( {a + b - 2} \right)}}{2}.\]Do $f$ là một song ánh từ $\left(\mathbb N^*\right)^2$ lên $\mathbb N^*$, nên ta có điều cần chứng minh.

PS. Bài này có thể giải đơn giản nữa, bằng cách chỉ rõ ra $a$ và $b$ qua hàm phần nguyên.
[RIGHT][I][B]Nguồn: MathScope.ORG[/B][/I][/RIGHT]
 
Thụy An is offline   Trả Lời Với Trích Dẫn
Old 20-09-2019, 01:22 AM   #29
Hải Thụy
Super Moderator
 
Tham gia ngày: Jan 2019
Bài gởi: 6
Thanks: 3
Thanked 2 Times in 2 Posts
Các bài toán Giải Tích
1.[ TPHCM Ngày 1] Cho dãy số $(u_n)$ xác định bởi: $$u_1\,=\,\frac{-1}{3},\,u_{n+1}\,=\,\frac{u_n\,+\,1}{\sqrt{{u_n}^2 \,+\,1}}\,-\,1,\,n\,=\,1,2,3...$$ Chứng minh dãy số có giới hạn và tìm giới hạn đó.
2. [ Lam Sơn-Thanh Hóa]Cho dãy $(x_n)$ xác định bởi: $$x_1\,=\,\alpha,\,(\alpha \in\,\mathbb{R}),\,\,x_{n+1}\,=\,(1\,+\,\frac{1}{n +1}\,-\,\frac{2}{(n+1)^2})x_n\,+\,\frac{8}{(n+1)^2},\,\, \forall n\,\geqslant\,1.$$ Tìm số hạng tổng quát của dãy $(x_n),$ từ đó tìm $\alpha$ để dãy $(x_n)$ có giới hạn hữu hạn.

3. [Lam Sơn-Thanh Hóa] Tìm tất cả các hàm $f:\,\mathbb{R}\,\to\,\mathbb{R},$ liên tục trên $\mathbb{R}$ thỏa: $$f(xy)\,+\,f(x+y)\,=\,f(xy+x)\,+\,f(y).$$
4. [Sóc Trăng ngày 1]Cho dãy số $(u_n)$ thỏa: $$\left\{ \begin{gathered} {u_1}\, = \,2020 \hfill \\ 2020{u_{n + 1}}\, = \,2019{u_n}\, + \,u_n^2,\,\,\,\,\,\forall \,n\, \in \,{N^*} \hfill \\ \end{gathered} \right.$$ Tính $\mathop {\lim }\limits_{n \to + \infty } \sum\limits_{k = 0}^n {\frac{{{u_k}}}{{{u_{k + 1}}\, - \,1}}.} $
5. [ Sóc Trăng ngày 2]Cho dãy số $(a_n)$ xác định như sau: $$\left\{ \begin{gathered} {a_1}\, = \,5,\,{a_2}\, = \,13 \hfill \\ {a_{n + 2}}\, = \,5{a_{n + 1}}\, - \,6{a_n},\,\,\,\,\,\forall \,n\, \in \,{N^*} \hfill \\ \end{gathered} \right.$$ Chứng minh rằng với $k$ nguyên dương bất kì, nếu $p$ là một ước nguyên tố của $a_{2k}\,+\,2.6^k$ thì $p$ cũng là ước của $a_{2k+1}\,+\,5.6^k.$ 6. [Lào Cai] Cho $(a_n),\,(b_n)$ thỏa: $$a_0\,=\,1,\,a_1\,=\,\frac{1}{2},\,b_n\,=\,\frac{ 1}{3}\,+\,2a_{n+1},\,2b_{n+1}\,=\,2b_n\,-\,a_n.$$ Với mỗi $n\,\in \mathbb{N},$ đặt ${c_n}\, = \,\frac{1}{{{2^{n + 1}}}}\sum\limits_{i = 0}^n {\frac{{{b_i}}}{{{a_i}}}.} $ Tính lim$c_n.$
7. [Cần Thơ]Cho hàm số $f$ liên tục trên $[0;2020]$ thỏa $f(2020)\,=\,f(0)\,+\,2020,$ $f(1010)\,\ne f(0)\,+\,1010.$ Chứng minh rằng tồn tại $x_1,\,x_2\,\in\, (0;2020)$ mà $x_1\,\ne\,x_2$ sao cho $f(x_1)\,-\,x_1\,=\,f(x_2)\,-\,x_2.$
8. [PTNK] Số thực $\alpha$ được gọi là điểm tụ của dãy số $(u_n)$ nếu tồn tại ít nhất một dãy con của dãy $(u_n)$ hội tụ đến $\alpha.$ $\,(a)\,$ Hãy chỉ ra một dãy có vô hạn điểm tụ. $\,(b)\,$ CMR nếu mọi dãy con của dãy $(u_n)$ đều hội tụ thì dãy $(u_n)$ cũng hội tụ. $\,(c)\,$ Gọi $S$ là tập các số chính phương. Dãy số $(a_n)$ thỏa $a_n\,=\,\frac{1}{n}$ nếu $n\,\in \,S$ bỏ ${0}$ và $a_n\,=\,\frac{1}{n^2}$ nếu $n\,\notin \,S.$ Xét tính hội tụ của các dãy $(a_n),\,(b_n)$ với $b_n\, = \,\sum\limits_{i = 0}^n {{a_i}.}$ 9. [Bắc Giang]Tìm hàm số liên tục: $f:\,\mathbb{R}\,\to\,\mathbb{R}$ thỏa: $$f(x+y)f(x-y)\,=\,f^2(x)f^2(y)\,\,\,\,\,\forall x,\,y\,\in\,\mathbb{R}.$$
10. [KHTN] Cho dãy số $(a_n)$ xác định bởi: $$a_0\,=\,1,\,a_1\,=\,\frac{1}{2},\,a_{n+1}\,=\,\f rac{{a_n}^3}{2{a_{n-1}}^2-{a_n}^2}$$ với mọi $n\,\geqslant \,1.$ Đặt ${x_n}\, = \,\sum\limits_{k = 1}^n {\frac{{{a_k}}}{{{2^k}}}.} $ CMR dãy $(x_n)$ hội tụ và tìm giới hạn đó.
11. [Hùng Vương-Bình Dương] Với mỗi $n\,\in\,\mathbb{N^*},$ đặt ${Q_n}(x)\, = \,\prod\limits_{i = 0}^n {(x - {i^2})} $ và kí hiệu ${Q'_n(x)}$ là đa thức đạo hàm của $Q_n(x).$ $\,1.\,$ Khi $n\,=\,2020$ thì đa thức $Q'_n(x)$ có bao nhiêu nghiệm thực? $\,2.\,$ CMR với mỗi $n$ nguyên dương, đa thức $Q'_n(x)$ có duy nhất một nghiệm thực $x_n$ thuộc $(0;1).$ $\,3.\,$ Tồn tại hay không giới hạn của dãy $(x_n)$ khi $n\,\to\,+\infty?$
12. [ĐH Vinh]Cho dãy $(x_n)$ xác định bởi: $$x_1\,=\,1,\,{x_{n + 1}}\, = \,\sqrt[3]{{8x_n^3\, + 12x_n^2 + 1}}$$ với mọi $n$ nguyên dương. Tính $\lim \frac{{{x_{n + 1}}}}{{{x_n}}}.$ 13. [Amsterdam]Cho hàm số $y\,=\,x\,+\,\frac{1}{x}$ với $x\,>\,0$ có đồ thị $(C).$ Một đường thẳng đi qua điểm $A(0;1)$ cắt $(C)$ tại hai điểm phân biệt $M$ và $N.$ Các tiếp tuyến với $(C)$ tại hai điểm phân biệt $M,\,N$ cắt nhau tại điểm $P.$ Tìm hoành độ điểm $P,$ và chứng minh rằng tung độ của điểm $P,$ kí hiệu $y_P$ thỏa $2\,<\,y_P\,<\,\frac{5}{2}.$
14. [Lâm Đồng] Cho số thực $a$ và dãy số thực $(u_n)$ thỏa: $$\left\{ \begin{gathered} {u_1}\, = \,a \hfill \\ {u_{n + 1}}\, = \,\ln (8\, + \,\cos {u_n}\, + \,\sin {u_n})\, + \,2019,\,\,\,\forall \,n\,=\,1,2,3...\,\,\, \hfill \\ \end{gathered} \right.$$ CMR dãy $(u_n)$ có giới hạn hữu hạn.
15. [Đồng Tháp]Cho dãy số thực dương $(x_n)$ xác định bởi: ${x_n}\, = \,2(1\, - \,\frac{1}{{2{n^2}\, + \sqrt {4{n^4}\, + \,1} }})$ với mọi $n$ nguyên dương. Đặt ${S_n}\, = \,\sum\limits_{i = 1}^n {\sqrt {{x_i}} } \,(n\, \geqslant \,1).$ $\,a.\,$ Tính $S_{20}.$ $\,b.\,$ CMR tồn tại vô hạn số nguyên dương $n$ sao cho $S_n$ nhận giá trị nguyên.
16. [Quảng Bình] Cho hai dãy số thực $(a_n),\,(b_n)$ thỏa mãn: $ \left\{ \begin{gathered} {a_1}\, = \,2 \hfill \\ {a_n}\, = \,2(n + {a_{n - 1}}),\,\forall \,n\, \geqslant 2 \hfill \\ \end{gathered} \right.$ và $\left\{ \begin{gathered} {b_1}\, = \,2 \hfill \\ {b_n}\, = \,\frac{{n + 1}}{{n - 1}}({b_1} + \,{b_2}\, + ... + \,{b_{n - 1}}),\,\forall \,n\, \geqslant 2 \hfill \\ \end{gathered} \right.$ $\,a.\,$ CMR $a_n\,<\,2^{n+2}$ với mọi $n$ nguyên dương.\\ $\,b.\,$ Tính giới hạn: $\lim \frac{{2({b_n} + n + 2) + {a_n}}}{{{6^n}}}.$
17. [Hà Tĩnh] Cho các dãy số $(u_n),\,(v_n)$ thỏa: $lim(u_n)^n\,=\,2,\,lim(v_n)^n\,=3;\,u_n,\,v_n\,\n eq \,1$ với mọi $n.$ $\,a.\,$ CMR lim$u_n\,=\,1.$ $\,b.\,$ Tìm lim$(\frac{2u_n+3v_n}{5})^n.$
18. [Hà Nam]
Cho hai số thực $a,\,b\,\in\,(0,1)$ và dãy số $(x_n)$ thỏa : $$x_1\,=\,a,\,x_2\,=\,b,\,{x_{n + 2}}\, = \,\frac{1}{4}x_{n + 1}^2\, + \,\frac{3}{4}\sqrt {{x_n}} \,,\,\forall \,n\, \geqslant 1.$$ CMR dãy $(x_n)$ có giới hạn hữu hạn và tính lim$x_n.$

19. [Phú Thọ] Có tồn tại hay không hàm $f(x)$ gián đoạn tại mọi điểm thuộc $\mathbb R$ còn $f(x)^2$ liên tục trên $\mathbb R$.
20. [Vĩnh Long] Cho dãy $(a_n)$ thỏa mãn điều kiện : $a_1\,=\,2$ và $(4-a_n)(6+a_{n-1})\,=\,24$ với mọi $n$ nguyên dương và $n\,\geqslant \,2.$
$\,a.\,$ CMR $a_n\,\neq \,0 $ với mọi $n\,\in\,\mathbb{N^*}.$
$\,b.\,$ Tính $S_{2020}\,=\,\frac{1}{a_1}+\frac{1}{a_2}+...+\fra c{1}{a_{2020}}.$
21. [Bình Dương-1] Cho dãy số $(u_n)$ thỏa $u_1\,=\,2020,\,u_{n+1}\,=\,\frac{{u^4}_n\,+\,2019 ^2}{{u^3}_n\,-\,u_n\,+\,4038}$ với mọi $n$ nguyên dương.
Đặt $v_n\,=\,\sum \limits_{k=1}^n{\frac{1}{{u^3}_k+2019}}$ với mọi $n\,\in\,\mathbb{N^*}.$ Tính lim$v_n.$
22. [KHTN-3] Cho dãy số $(a_n)$ thỏa: $a_1\,=\,\frac{2}{3},\,a_{n+1}\,=\,\frac{{a_n}^2+( n-1)a_n+2}{n+2}$ với mọi $n$ nguyên dương.
$\,a.\,$ CMR dãy $(a_n)$ bị chặn.
$\,b.\,$ Cm dãy $(a_n)$ hội tụ và tìm giới hạn.
[RIGHT][I][B]Nguồn: MathScope.ORG[/B][/I][/RIGHT]
 

thay đổi nội dung bởi: Hải Thụy, 22-09-2019 lúc 03:06 PM
Hải Thụy is offline   Trả Lời Với Trích Dẫn
The Following User Says Thank You to Hải Thụy For This Useful Post:
MATHSCOPE (20-09-2019)
Old 20-09-2019, 03:05 PM   #30
Thụy An
+Thành Viên+

 
Tham gia ngày: Oct 2017
Bài gởi: 93
Thanks: 1
Thanked 68 Times in 45 Posts
Trích:
Nguyên văn bởi MATHSCOPE View Post
[Phú Thọ.] Tìm các số tự nhiên $k,\,m,\,n$ sao cho \[k^3=5^m+7^n.\]
Ta thấy $k$ chẵn nên $5^m+7^n$ là bội của $4$, kết hợp $5\equiv 1\mod 4$ vì thế $n>0$. Để ý rằng, $5$ là căn nguyên thủy mod $7$, cho nên từ $7\nmid k$ có $5^{2m}\equiv k^6\mod 7$ tức là $3\mid m$, viết $m=3t,\,t\in\mathbb N$ và đặt $5^t=l$ ta có $7\nmid kl$ và\[\left( {k - l} \right)\left( {{k^2} + kl + {l^2}} \right) = {7^n}.\]Như vậy, $\gcd\left(k-l,\,k^2+kl+l^2\right)$ là ước của $7^n$ mà $7\nmid 3kl$ và\[\gcd \left( {k - l,{\mkern 1mu} {k^2} + kl + {l^2}} \right) = \gcd \left( {k - l,{\mkern 1mu} 3kl} \right).\]Từ đây có $k-l=1$, kéo theo\[3{k^2} - 3k + 1 = {7^n}.\]Nếu $m\ge 1$, có $k=l+1\equiv 1\mod 5$, vì $7$ là căn nguyên thủy mod 5 nên ta có $n=4s$ với $s\in\mathbb N^*$, từ đây có luôn mâu thuẫn là \[{k^3} = {5^m} + {7^{4s}} \equiv 2\;\;\,\left( {\bmod 4} \right).\]Vậy, $m=0$ tức là $l=1$ từ đó $k=2$ và $n=1$.

------------------------------
Trích:
Nguyên văn bởi Hải Thụy View Post
19. [Phú Thọ] Có tồn tại hay không hàm $f(x)$ gián đoạn tại mọi điểm thuộc $\mathbb R$ còn $f(x)^2$ liên tục trên $\mathbb R$.
Xét hàm số\[f\left( x \right) = \left\{ \begin{array}{l}
1,&\:{\mkern 1mu} {\rm{nếu}}\:x \in \mathbb Q,\\
- 1,&\:{\mkern 1mu} {\rm{nếu}}\:x \in \mathbb R \setminus \mathbb Q.
\end{array} \right.\]
Hàm này thỏa mãn yêu cầu.
[RIGHT][I][B]Nguồn: MathScope.ORG[/B][/I][/RIGHT]
 

thay đổi nội dung bởi: Thụy An, 20-09-2019 lúc 03:47 PM Lý do: Tự động gộp bài
Thụy An is offline   Trả Lời Với Trích Dẫn
Trả lời Gởi Ðề Tài Mới

Bookmarks

Ðiều Chỉnh
Xếp Bài

Quuyền Hạn Của Bạn
You may not post new threads
You may not post replies
You may not post attachments
You may not edit your posts

BB code is Mở
Smilies đang Mở
[IMG] đang Mở
HTML đang Tắt

Chuyển đến


Múi giờ GMT. Hiện tại là 09:19 AM.


Powered by: vBulletin Copyright ©2000-2024, Jelsoft Enterprises Ltd.
Inactive Reminders By mathscope.org
[page compression: 120.50 k/136.68 k (11.84%)]